Question

7b. Consider three stocks A, B, and C, with closing price at time t being P, and the number of shares time t as Q Stock C spl
0 0
Add a comment Improve this question Transcribed image text
Answer #1

Part (i)

Index0 = (30 + 62 + 98) / 3 = 63.33

Index1 = (35 + 56 + 102) / 3 = 64.33

hence, return on index = (64.33 - 63.33) / 63.33 = 1.58%

Part (ii)

Return on A = 35/30 - 1 = 16.67%

Return on B = 56 / 62 - 1 = - 9.68%

Return on C = 102 / 98 - 1 = 4.08%

Average of these returns = 3.69%

Part (iii)

P0 Q0 P0 x Q0 P1 Q1 P1 x Q1
A             30        1,000           30,000             35        1,000           35,000
B             62        5,000        310,000             56        5,000        280,000
C             98        3,000        294,000           102        3,000        306,000
Total        634,000        621,000

Return on value weighted index = 621,000 / 634,000 - 1 = - 2.05%

Add a comment
Know the answer?
Add Answer to:
7b. Consider three stocks A, B, and C, with closing price at time t being P, and the number of shares time t as Q S...
Your Answer:

Post as a guest

Your Name:

What's your source?

Earn Coins

Coins can be redeemed for fabulous gifts.

Not the answer you're looking for? Ask your own homework help question. Our experts will answer your question WITHIN MINUTES for Free.
Similar Homework Help Questions
  • Consider the three stocks in the following table. Pt represents price at time t,

    Consider the three stocks in the following table. Pt represents price at time t, and Qt represents shares outstanding at time t. Stock C splits two-for-one in the last period.a) Calculate the rate of return on a price-weighted index of the three stocks for the first period (from t=0 to t=1). b) What must happen to the divisor for the price-weighted index in year 2? c) Calculate the rate of return of the price-weighted index for the second period (from t =...

  • 9. Consider the three stocks in the following table. P, represents price at time t, and...

    9. Consider the three stocks in the following table. P, represents price at time t, and Q, represents shares outstanding at time t. Stock C splits two-for-one in the last period. (LO 2-2) Pt 95 45 110 100 200 200 100 200 400 95 90 50 100 100 200 200 45 a. Calculate the rate of return on a price-weighted index of the three stocks for the first period (i 0 to t 1). b. What must happen to the...

  • Consider the three stocks in the following table. Pt represents price at time, and O+ represents...

    Consider the three stocks in the following table. Pt represents price at time, and O+ represents shares outstanding at time i Stock splits two-for-one in the last period. Po P2 lo 100 200 200 P1 101 51 122 01 100 101 02 100 200 400 200 51 C 112 200 61 a. Calculate the rate of return on a price-weighted index of the three stocks for the first period (1=0 to r= 1). (Do not round intermediate calculations. Round your...

  • Consider the three stocks in the following table. Prepresents price at time, and C splits two...

    Consider the three stocks in the following table. Prepresents price at time, and C splits two for one in the last period represents shares outstanding at time t. Stock A B C PO 130 120 240 DO 135 270 270 PL 140 115 250 O 135 270 270 P2 140 115 125 02 135 270 540 Calculate the first-period rates of return on the following indexes of the three stocks (t = 0 tot = 1): (Do not round intermediate...

  • Consider the three stocks in the following table. Pt represents price at time t, and Qt...

    Consider the three stocks in the following table. Pt represents price at time t, and Qt represents shares outstanding at time t Stock C splits two-for-one in the last period. Po 96 56 112 @o P1 100 101 20051 200 122 01 100 200 200 P2 101 51 61 02 100 200 Calculate the first-period rates of return on the following indexes of the three stocks: (Do not round Intermediate calculations. Round your answers to 2 decimal places.) a. A...

  • the three stocks in the following table. Pe represents price at time t, and Q represents...

    the three stocks in the following table. Pe represents price at time t, and Q represents shares outstanding at time t Stock C splits two for one in the last period. Po A 75 35080 350 80350 B 55 800 50 800 50 800 С 60 900 70 900 351,800 o. Calculate the rate of return on a price weighted index of the three stocks for the first period (t-0 to t-1) (Do not round intermediate calculations. Round your answer...

  • Consider the three stocks in the following table. Pt represents price at time t, and Qt...

    Consider the three stocks in the following table. Pt represents price at time t, and Qt represents shares outstanding at timet. Stock C splits two-for-one in the last perioc p. 90 50 200 45 200 45 200 95 100 95 100 100 200 1020055 Calculate the first-period rates of return on the following indexes of the three stocks: (Do not round intermediate calculations.Round answers to 2 decimal places.) a. A market value-weighted index Rate of return b. An equally weighted...

  • Consider the three stocks in the following table. P represents price at time t, and Q,...

    Consider the three stocks in the following table. P represents price at time t, and Q, represents shares outstanding at time t.Stock C splits two-for-one in the last period 86 100 91 100 91 100 92 200 102 200 51 400 200 4 41 a ulate the rate of re u on a price-weighted index o the tree stocks for the f st pero ล. O to t-1 Do not round intermediate calculations. Round your answer to 2 decimal Rate...

  • Consider the three stocks in the following table. Pt represents price at time t, and Qt...

    Consider the three stocks in the following table. Pt represents price at time t, and Qt represents shares outstanding at time t. Stock C splits two-for-one in the last period. P0 Q0 P1 Q1 P2 Q2 A 100 100 105 100 105 100 B 60 200 55 200 55 200 C 120 200 130 200 65 400 a. Calculate the rate of return on a price-weighted index of the three stocks for the first period (t = 0 to t...

  • Consider the three stocks in the following table. Pt represents price at time t, and Qt...

    Consider the three stocks in the following table. Pt represents price at time t, and Qt represents shares outstanding at time t. Stock C splits two-for-one in the last period. P0 Q0 P1 Q1 P2 Q2 A 88 100 93 100 93 100 B 48 200 43 200 43 200 C 96 200 106 200 53 400 a. Calculate the rate of return on a price-weighted index of the three stocks for the first period (t = 0 to t...

ADVERTISEMENT
Free Homework Help App
Download From Google Play
Scan Your Homework
to Get Instant Free Answers
Need Online Homework Help?
Ask a Question
Get Answers For Free
Most questions answered within 3 hours.
ADVERTISEMENT
ADVERTISEMENT
ADVERTISEMENT